U of I Mock Putnam Exam SEPT. 29, 2009 Solutions

Download as pdf or txt
Download as pdf or txt
You are on page 1of 3

U OF I MOCK PUTNAM EXAM

SEPT. 29, 2009


Solutions

1. Suppose P (x) is a polynomial with integer coefficients such that none of the values
P (1), . . . , P (2009) is divisible by 2009. Prove that P (n) 6= 0 for all integers n.
Solution. We use the fact that, for any integers a, b and m 1, a b mod m
implies P (a) P (b) mod m. This follows from the general properties of congruences.
Given an integer n, let r {1, 2, . . . , 2009} be such that n r mod 2009. Then, by
the above fact and the given hypothesis, P (n) P (r) 6 0 mod 2009, and hence
P (n) 6= 0.

2. Find a function f (x) that satisfies, for all x 0,


sZ
x
f (x) = (f (t)2 + f 0 (t)2 )dt + 2009.
0

Solution. [Problem B1, Putnam 1990] Square the given equation and differentiate
to get
2f (x)f 0 (x) = f (x)2 + f 0 (x)2 ,
or
(f (x) f 0 (x))2 = 0,
which is equivalent to f 0 (x) = f (x). The latter equation has solution f (x) = Cex ,
where C is an arbitrary constant. Substituting this into the original equation, we get
s Z
x p
Cex = C 2 2(Cet )2 dt + 2009 = C 2 e2x C 2 + 2009,
0

and choosing C = 2009, we see that this equation is satisfied. Thus, f (x) = 2009ex
is the desired solution.

3. Let A = (a1 , a2 , a3 , . . .) be a permutation of the positive integers. (In other words, ak


is a positive integer for each k, and for each positive integer n, there exists exactly
one k such that ak = n.) Prove that A contains a triple (ai , aj , ak ) with i < j < k
and aj ai = ak aj = d > 0.
Solution. Let j be the least integer such that aj > a1 , and let k be the unique
index such that ak = 2aj a1 . Since 2aj a1 > aj > a1 , it follows from the definition
of the index j that k > j. The triple (a1 , aj , ak ) has the required properties.
4. Let b1 , . . . , bn be integers greater than 1, and let B = b1 . . . bn denote their product.
Let di be the number of digits of B expressed in base bi . For example, if b1 = 10,
b2 = 3, b3 = 2, then B = 10 3 2 = (60)10 = (2020)3 = (111100)2 , so d1 = 2, d2 = 4,
d3 = 6. Show that
d1 + + dn > n2 .

Solution. First note that a positive integer m expressed in a base b 2 has d digits
if and only if bd1 m < bd , or equivalently, d 1 ln m/ ln b < d.
Now, set i = ln bi and note that
n
X n
X
ln B = ln(b1 . . . bn ) = ln bj = j .
j=1 i=j

By the above inequality,


n
ln B 1 X
di > = j .
ln bi i
j=1

Summing over i = 1, 2, . . . , n and using the arithmetic-geometric mean inequality


twice, we get

n n
! n
X X 1 X
di > j

i=1 i=1 i j=1

 1/n X n
1 1
n ... j
1 n
j=1

n
!1 n
1 X X
n i j = n2 .
n
i=1 j=1

Note: Instead of two applications of the AGM inequality, we could have used the
arithmetic-harmonic mean inequality:
n
! n
!1
1 X 1X 1
i .
n n i
i=1 i=1

A third method is to use Cauchys inequality in the form


n
!2 n
! n !
2
X 1/2 1/2
X 1 X
n = i i i .
i
i=1 i=1 i=1

5. Show that any positive integer containing exactly 2009 digits (in decimal), none of
whose digits is zero, is either divisible by 2009, or can be changed to an integer that
is divisible by 2009 by replacing some, but not all, of its digits by 0.

1
Solution. Let n = a1 a2 . . . a2009 be the given integer, let n0 = 0, and for k =
1, 2, . . . , 2009 let nk denote the number obtained by replacing all but the first k digits
of n by the digit 0, i.e., nk = a1 a2 . . . ak 00 . . . 0. By the pigeon hole principle, two of
the 2010 numbers n0 , n1 , . . . , n2009 , say nk1 and nk2 with k2 > k1 , must be congruent
modulo 2009. It follows that the difference nk2 nk1 is divisible by 2009. Now nk2 nk1
is the number obtained from n by replacing the first k1 and the last (2009 k2 ) digits
by 0. Since k2 > k1 , we have k1 + (2009 k2 ) < 2009, so not all of the 2009 digits are
replaced by zero. Thus, the number nk2 nk1 has the required properties.

6. Let A be an infinite set of positive integers, and let A(n) denote the number of
elements of A that are n. Suppose that the series
X1
a
aA

converges. Show that limn A(n)/n = 0.


Solution. [Problem B5, Putnam 1969] We argue by contradiction. Suppose A(n)/n
does not tend to 0. Then there exists  > 0 and an infinite sequence n1 < n2 < of
positive integers such that A(nk )/nk  for all k. Let N be a fixed positive integer.
Since nk as k , we have nk > N for all large enough k. For such k we have
X 1 X 1 A(nk ) A(N ) A(N )
 .
a nk nk nk
aA aA
N <ank N <ank

Letting k , we conclude
X 1
.
a
aA,a>N
P
Since N was arbitrary, the series aA 1/a cannot be convergent.

You might also like